11
$\begingroup$

My guess is that there exists a constant $C$ such that $A(X) \sim C (\log X)^2$.

$\endgroup$
3

2 Answers 2

28
$\begingroup$

$$ \sum_{1 \leq i,j \leq X} \frac{1}{\mathrm{lcm}(i,j)} = \sum_{1 \leq i,j \leq X} \frac{\mathrm{gcd}(i,j)}{ij} $$ $$ = \sum_{1 \leq i,j \leq X} \frac{\sum_{d|i,j} \phi(d)}{ij}$$ $$ = \sum_{d \leq X} \phi(d) \sum_{1 \leq i,j \leq X: d|i,j} \frac{1}{ij}$$ $$ = \sum_{d \leq X} \frac{\phi(d)}{d^2} \sum_{1 \leq i',j' \leq X/d} \frac{1}{i'j'}$$ $$ = \sum_{d \leq X} \frac{\phi(d)}{d^2} ( \sum_{1 \leq i \leq X/d} \frac{1}{i})^2$$ $$ = \sum_{d \leq X} \frac{\phi(d)}{d^2} ( \log(X/d) + O(1))^2$$ $$ = \sum_{d \leq X} \frac{\phi(d)}{d^2} ( \log^2(X) - 2 \log(d) \log(X) + \log^2(d) + O( \log X ) )$$ $$ = A_0 \log^2 X - 2 A_1 \log X + A_2 + O( \log^2 X )$$

where $$ A_j := \sum_{d \leq X} \frac{\phi(d) \log^j d}{d^2}.$$

One can compute asymptotics for the $A_j$ by Perron's formula, but we proceed instead by elementary means. Since $\phi(d) = \sum_{d=ab} \mu(a) b$ we have $$ A_j = \sum_{ab \leq X} \frac{\mu(a) b \log^j(ab)}{a^2b^2}$$ $$ = \sum_{a \leq X} \frac{\mu(a)}{a^2} (\frac{\log^{j+1}(X)}{j+1} + O( (1 + \log^j(a)) \log^j X ) )$$ $$ = \frac{1}{j+1} \log^{j+1} X \sum_{a \leq X} \frac{\mu(a)}{a^2} + O(\log^j X)$$ $$ = \frac{1}{(j+1)\zeta(2)} \log^{j+1} X + O(\log^j X).$$ Hence $$ \sum_{1 \leq i,j \leq X} \frac{1}{\mathrm{lcm}(i,j)} = \frac{1}{3 \zeta(2)} \log^3 X + O(\log^2 X).$$

$\endgroup$
3
  • 2
    $\begingroup$ Your estimate for $A_0$ leads to division by $0$. I think you mean to have $A_j = \frac{1}{(j+1)\zeta(2)} \log^{j+1} X + O(\log^j X)$. $\endgroup$
    – Greg Hurst
    Aug 12, 2020 at 21:52
  • $\begingroup$ Corrected, thanks - T. $\endgroup$
    – Terry Tao
    Aug 12, 2020 at 23:23
  • 6
    $\begingroup$ This result also appears (with a secondary main term) in Theorem 7.1 of cs.uwaterloo.ca/journals/JIS/VOL10/Bordelles2/bordelles61.pdf $\endgroup$
    – GH from MO
    Aug 13, 2020 at 2:44
10
$\begingroup$

If we are talking about "elementary", then just multiply the original sum by the sum of inverse squares and note that if we have three numbers $a=a'd, b=b'd, n^2$ where $(a',b')=1$, $d,n$ are arbitrary, then $A=a'n, B=b'n, d$ are arbitrary $3$ integers with the product $LCM(a,b)n^2$. The original triple sum has the bounds $a'd, b'd\le X$, $n$ formally unrestricted, but restricting it to $[1,N]$ for fixed $N$ changes the triple sum $1+O(1/N)$ times. But then we can squeeze the bounds on $A,B,d$ between $Ad\le X, Bd\le X$ (treating the implicit $n=(A,B)$ as unrestricted) to get the lower bound and $Ad\le NX, Bd\le NX$ (assuming $n\le N$ now) to get the upper bound, so we get an answer for the triple sum of $\frac 1{ABd}$ between roughly speaking $\frac13\log^3 X$ and $\frac13\log^3(XN)$ with additive error $O(\log^2(XN))$ (no number theory in this sum!), which have the same asymptotic up to $1+O(\frac{\log N}{\log X})$. Taking $N$ about $\sqrt{\log X}$, we get the total multiplicative error $1+O(\frac{1}{\sqrt{\log X}})$, which is, of course, suboptimal but who cares. :-)

$\endgroup$

Your Answer

By clicking “Post Your Answer”, you agree to our terms of service and acknowledge you have read our privacy policy.

Not the answer you're looking for? Browse other questions tagged or ask your own question.